ChaseDream
搜索
返回列表 发新帖
00:00:00

In Gandania, where the government has a monopoly on tobacco sales, the incidence of smoking-related health problems has risen steadily for the last twenty years. The health secretary recently proposed a series of laws aimed at curtailing tobacco use in Gandania. Profits from tobacco sales, however, account for ten percent of Gandania's annual revenues. Therefore, Gandania cannot afford to institute the proposed laws.

Which of the following, if true, most seriously weakens the argument?

正确答案: A

更多相关帖子

524

帖子

15

好友

4712

积分

ChaseDream

注册时间
2003-03-17
精华
8
解析
查看: 2066|回复: 5
打印 上一主题 下一主题

问一道真题(取自GMAT30回)

[复制链接]
楼主
发表于 2005-9-29 22:31:00 | 只看该作者

问一道真题(取自GMAT30回)

In Gandania, where the government has a monopoly on tobacco sales, the incidence of smoking-related health problems has risen steadily for the last twenty years. The health secretary recently proposed a series of laws aimed at curtailing tobacco use in Gandania. Profits from tobacco sales, however, account for ten percent of Gandania's annual revenues. Therefore, Gandania cannot afford to institute the proposed laws.



Which of the following, if true, most seriously weakens the argument?


A)All health care in Gandania is government-founded.


B)Implementing the proposed laws is not likely to cause a significant increase in the amount of tobacco Gandania exports.


C)The percentage of revenue Gandania receives from tobacco sales has remained steady in recent years.


D)Profits from tobacco sales far surpass any other single source of revenue for the Gandania government.


E)No government official in Gandania has ever previously proposed laws aimed at curtailing tobacco use.


正确答案: A


实在想不出来为什么A是攸关选项又是正解耶…



求助!!!



沙发
发表于 2005-9-29 22:43:00 | 只看该作者

因为医疗机构是goverment-founded,所以虽然政府在烟草上赚钱,但医疗方面支出增加了

板凳
发表于 2005-9-29 23:24:00 | 只看该作者

In Gandania, where the government has a monopoly on tobacco sales.也就是说,收入是政府的,并且占其利润的10%。如果 a series of laws aimed at curtailing tobacco use in Gandania,那么随着销售的下降,政府的烟草销售收入下降。但是,烟草销售下降,抽烟的人少了,the incidence of smoking-related health problems has decrease,而如果如A)All health care in Gandania is government-founded,那么医疗支出也会减少。


所以,削弱结论Therefore, Gandania cannot afford to institute the proposed laws.

地板
 楼主| 发表于 2005-9-30 20:24:00 | 只看该作者

Soga…谢谢楼上,讲解的很清楚


只是,这个答案似乎需要一点点想像力


文中没有提到任何关于"医疗机构须负担医疗费用"呀,


只说因为越来越多抽烟导致的健康问题,故此law的目的是想减少增加的健康问题,(或许只是基于关心人民身体健康)…那么即使是由政府创办也是基于政府关心人民啊…??



很笨的问

5#
发表于 2005-9-30 20:34:00 | 只看该作者

欧觉得,要是问题改成assumption,是不是A就好理解了?

6#
发表于 2005-9-30 21:13:00 | 只看该作者
以下是引用dearpiow在2005-9-30 20:24:00的发言:

Soga…谢谢楼上,讲解的很清楚



只是,这个答案似乎需要一点点想像力



文中没有提到任何关于"医疗机构须负担医疗费用"呀,



只说因为越来越多抽烟导致的健康问题,故此law的目的是想减少增加的健康问题,(或许只是基于关心人民身体健康)…那么即使是由政府创办也是基于政府关心人民啊…??






很笨的问



做逻辑题,最重要的是抓住推理链和结论的具体性和特殊性。


建议MM看看lawyer-1 NN的有关论述。仔细体会,一定会大有收获。

您需要登录后才可以回帖 登录 | 立即注册

Mark一下! 看一下! 顶楼主! 感谢分享! 快速回复:

手机版|ChaseDream|GMT+8, 2024-9-17 10:37
京公网安备11010202008513号 京ICP证101109号 京ICP备12012021号

ChaseDream 论坛

© 2003-2023 ChaseDream.com. All Rights Reserved.

返回顶部